Wat is the remainder when a +ve integer n is divided by 6???

This topic has expert replies
Junior | Next Rank: 30 Posts
Posts: 10
Joined: Thu Oct 24, 2013 10:05 pm
I came across this question while trying the GMAT prep
Question: What is the remainder when a positive integer n is divided by 6?
(1)n is a multiple of 5
(2)n is a multiple of 12

Answer: B

Can somebody pls explain how the answer is derived?

My understanding of the question is if we can atleast narrow it down to a range of numbers which satisfy the condition of being divisible by 6 it is sufficient to answer the question, need not specifically find the remainder.

The first statement tells n is a multiple of 5, thus the range could be 5,10,15,20,25,30,35,40,45,50,55,60....Of this range, checking 10 for divisibility with 6, the remainder is 4.Similarly for 15, it is 3 and so on. Thus I am able to tell what the remainder is.

The second statement tells n is a multiple of 12, thus the range could be 12,24,36,48,60....Of this range, checking 12 for divisbilty with 6, the remainder is 0,Similarly for 24, again remainder is 0 and so on. Thus again I am able to tell what the remainder is.

So according to me the answer should be D (Each statement alone is sufficient).
But the answer given is B.
Had the question been what is the value of n , then B would have been the right answer.

Please help me in understanding what is wrong in my logic. Btw this type of question falls in which range like 300-500/600-700??

User avatar
GMAT Instructor
Posts: 15539
Joined: Tue May 25, 2010 12:04 pm
Location: New York, NY
Thanked: 13060 times
Followed by:1906 members
GMAT Score:790

by GMATGuruNY » Fri Nov 29, 2013 3:44 am
sbhawal wrote:I came across this question while trying the GMAT prep
Question: What is the remainder when a positive integer n is divided by 6?
(1)n is a multiple of 5
(2)n is a multiple of 12
The question stem asks for a SPECIFIC VALUE (the remainder when n is divided by 6).
One way to evaluate the statements is to test cases.
If the value is THE SAME in every case. then the statement is SUFFICIENT.
If the value is NOT THE SAME in every case, then the statement is INSUFFICIENT.

Statement 1:
Options for n = 5, 10, 15, 20...
When 10 is divided by 6, the remainder is 4.
When 15 is divided by 6, the remainder is 3.
Since the remainder can be different values, INSUFFICIENT.

Statement 2:
Options for n = 12, 24, 36, 48...
When 12 is divided by 6, the remainder is 0.
When 24 is divided by 6, the remainder is 0.
When 36 is divided by 6, the remainder is is 0.
Since the remainder is the same in every case, SUFFICIENT.

The correct answer is B.
Private tutor exclusively for the GMAT and GRE, with over 20 years of experience.
Followed here and elsewhere by over 1900 test-takers.
I have worked with students based in the US, Australia, Taiwan, China, Tajikistan, Kuwait, Saudi Arabia -- a long list of countries.
My students have been admitted to HBS, CBS, Tuck, Yale, Stern, Fuqua -- a long list of top programs.

As a tutor, I don't simply teach you how I would approach problems.
I unlock the best way for YOU to solve problems.

For more information, please email me (Mitch Hunt) at [email protected].
Student Review #1
Student Review #2
Student Review #3

User avatar
Master | Next Rank: 500 Posts
Posts: 490
Joined: Thu Jul 04, 2013 7:30 am
Location: Chennai, India
Thanked: 83 times
Followed by:5 members

by Uva@90 » Fri Nov 29, 2013 6:09 am
Hi Sbhawal
My understanding of the question is if we can atleast narrow it down to a range of numbers which satisfy the condition of being divisible by 6 it is sufficient to answer the question, need not specifically find the remainder.

The first statement tells n is a multiple of 5, thus the range could be 5,10,15,20,25,30,35,40,45,50,55,60....Of this range, checking 10 for divisibility with 6, the remainder is 4.Similarly for 15, it is 3 and so on. Thus I am able to tell what the remainder is.


What you did is correct,
But, Thing you have to note is,
You got remainder 4 when you divide 10 by 6
and remainder 3 divide 15 by 6

Since remainder is changing.
Statement 1 is Insufficient.

Hope it helps you.

Regards,
Uva.
Known is a drop Unknown is an Ocean

GMAT/MBA Expert

User avatar
GMAT Instructor
Posts: 16207
Joined: Mon Dec 08, 2008 6:26 pm
Location: Vancouver, BC
Thanked: 5254 times
Followed by:1268 members
GMAT Score:770

by Brent@GMATPrepNow » Fri Nov 29, 2013 7:52 am
sbhawal wrote: What is the remainder when a positive integer n is divided by 6?
(1)n is a multiple of 5
(2)n is a multiple of 12

Btw this type of question falls in which range like 300-500/600-700??
When posting questions, please use the spoiler function to hide the correct answer. This will allow others to attempt the question without seeing the final answer.

I'd say this questions falls in the 300 - 400 range.


Target question: What is the remainder when a positive integer n is divided by 6?

Statement 1: n is a multiple of 5
There are several values of n that satisfy this condition. Here are two:
Case a: n = 5, in which case the remainder is 5, when n is divided by 6
Case b: n = 10, in which case the remainder is 4, when n is divided by 6
Since we cannot answer the target question with certainty, statement 1 is NOT SUFFICIENT

Statement 2: n is a multiple of 12
In other words, n = 12k for some integer k
We can rewrite this as n = (6)(2)k for some integer k
From this, we can see that n is a multiple of 6, which means the remainder must equal 0, when n is divided by 6
Since we can answer the target question with certainty, statement 2 is SUFFICIENT

Answer = B

Cheers,
Brent
Brent Hanneson - Creator of GMATPrepNow.com
Image

Junior | Next Rank: 30 Posts
Posts: 10
Joined: Thu Oct 24, 2013 10:05 pm

by sbhawal » Fri Nov 29, 2013 8:49 am
Ohk! Now I get it, the trick is about getting same value every time!!

Thanks!
Uva@90 wrote:Hi Sbhawal
My understanding of the question is if we can atleast narrow it down to a range of numbers which satisfy the condition of being divisible by 6 it is sufficient to answer the question, need not specifically find the remainder.

The first statement tells n is a multiple of 5, thus the range could be 5,10,15,20,25,30,35,40,45,50,55,60....Of this range, checking 10 for divisibility with 6, the remainder is 4.Similarly for 15, it is 3 and so on. Thus I am able to tell what the remainder is.


What you did is correct,
But, Thing you have to note is,
You got remainder 4 when you divide 10 by 6
and remainder 3 divide 15 by 6

Since remainder is changing.
Statement 1 is Insufficient.

Hope it helps you.

Regards,
Uva.
:D